Solve Iterated Integral Using Fundamental Theorem & Fubini's

  • Thread starter Thread starter skaterbasist
  • Start date Start date
  • Tags Tags
    Integrals
Click For Summary
The discussion focuses on evaluating an iterated integral using the Fundamental Theorem of Calculus and Fubini's Theorem. The user initially struggles with the integration process, specifically integrating with respect to y and then x, leading to an incorrect answer. After some back-and-forth, it is revealed that the mistake was in differentiating instead of integrating during the calculation. The user ultimately confirms that their initial integration was correct, which resolved their confusion. The thread highlights the importance of careful application of integration techniques in solving iterated integrals.
skaterbasist
Messages
13
Reaction score
0

Homework Statement



Evaluate the iterated integral [URL]https://webwork.csun.edu/webwork2_files/tmp/equations/e4/1efafbd0e820388c5c73acc695601b1.png[/URL]

Homework Equations



Fundamental Theorem of Calculus & Fubini's Theorem

The Attempt at a Solution



I have been working on this problem for the last hour and haven't been able to solve it thus far. I integrated the inside of the integral with respect to y from 4 to 3, and then integrated the result from the first integral with respect to x from 2 to 1. In the process, I used substitution to solve the integrals.

The answer I keep on getting is (1/(4(3x+4)^4)) - (1/(4(3x+3)^4)) and solve from x= 2 to 1 which results in a small fractional answer of 7.5674005*10^(-5). But my online program keeps on saying its incorrect.

Can anyone give me a heads up as to where I am going wrong? This is one of the few iterated integrals that is giving me problems right now :(
 
Last edited by a moderator:
Physics news on Phys.org
Its better to post your work.
Here is a start:
the integration of 1/(3x+y)^2 with respect to y from y=3 to y=4 is :
(-1/3x+4) - (1/3x+3)
and this result is easy to integrate with repsect to x, isn't it ?
 
I found out what my problem was. I made the stupid mistake of differentiating u^-2 when I should have integrated. Thank you. Your first integration was correct, and that was what was causing my mistake.
 
Question: A clock's minute hand has length 4 and its hour hand has length 3. What is the distance between the tips at the moment when it is increasing most rapidly?(Putnam Exam Question) Answer: Making assumption that both the hands moves at constant angular velocities, the answer is ## \sqrt{7} .## But don't you think this assumption is somewhat doubtful and wrong?

Similar threads

  • · Replies 10 ·
Replies
10
Views
2K
Replies
3
Views
2K
Replies
9
Views
2K
  • · Replies 105 ·
4
Replies
105
Views
6K
Replies
7
Views
2K
  • · Replies 15 ·
Replies
15
Views
2K
  • · Replies 3 ·
Replies
3
Views
2K
  • · Replies 5 ·
Replies
5
Views
3K
  • · Replies 12 ·
Replies
12
Views
2K
  • · Replies 7 ·
Replies
7
Views
2K